Adult Exam 3

¡Supera tus tareas y exámenes ahora con Quizwiz!

What are the risk factors that contribute to the development to CKD?

Glomerular disease or glomerulonephritis- uncontrolled and unregulated inflammation in the glomeruli two kinds infections and hereditary. Nephritis diseases-hematuria is the hallmark (lupus, IgA nephropathy, post strep infection) Nephrotic diseases-proteinuria is the hallmark-primary amyloidosis.

What is one of the manifestations of a middle cerebral artery stroke?

Greater weakness of the upper extremity, shoulder rotates internally, hip rotates externally, affected foot is plantar flexed and inverted. This is related to nerve damage.

What arises from an injury to the glomerular filtration barrier?

Hematuria, results in the passage of red blood cells into the urinary space which causes oxidative stress, inflammation, and structural damage to the kidney.

Stroke manifestations for the posterior cerebral vertebral artery:

Hemianopsia, visual hallucination, spontaneous pain, motor deficit. Cranial nerve deficits, diplopia, dizziness, nausea, vomiting, dysarthria, dysphagia, and or coma.

What are the primary causes of adrenal insufficiency?

Hemorrhage Tuberculosis Acquired immune deficiency syndrome

What are some complications of peritoneal dialysis?

Hernias from increased intrabdominal pressure Lower back problems-increased pressure Bleeding-with catheter placement Pulmonary complications-decreased lung expansion, atelectasis, pneumonia, bronchitis. Protein loss

What is an absence (petit mal) seizure?

Usually lasts <1 minute. Involves staring into space, brief loss of consciousness, rhythmic blinking of the eye, lip-smacking Most often occur in children. May occur in clusters and cause a brief loss of consciousness.

What are some Musculoskeletal manifestations that accompany CKD?

Vascular and soft tissue calcifications Osteomalacia Osteitis fibrosa

What are some indications for renal replacement therapy when conservative therapy is not effective for AKI?

Volume overload Elevated serum potassium level Metabolic acidosis BUN level less than 120 Significant change in mental status Pericarditis, pericardial effusion or cardiac tamponade clinical status of patient

What is uremia?

When GFR is less than 15 mL/min a syndrome in which kidney function declines to the point that symptoms occur in multiple body systems Manifestations: vary on cause, comorbidities, age, adherence to medical regime.

When are temporary vascular assess used for dialysis?

When waiting for AVF or if other forms have failed. Exit on upper chest, and tunneled to internal or external jugular vein tip is in the right atrium one or two cuffs that prevent infection and anchor catheter.

What is aphasia?

loss of ability to understand or express speech, caused by brain damage.

What is dysuria?

painful or difficult urination-UTI, interstitial cystitis, urethral stones, and a variety of pathological conditions.

What are the non modifiable causes of CKD?

Ethnicity Premature birth Age Family history Diseases-polycystic kidney disease Other diseases that predispose a person to renal disease-good pasture's disease, SLE

What are some neurologic manifestations that accompany CKD?

Fatigue Headache Sleep disturbance Encephalopathy

What is anuria?

Less than 100 mL of urine in 24 hours- acute kidney injury, end stage renal disease, bilateral ureteral obstruction, urethral irritation, UTI, urethral stones.

What are some pumonary manifestations that accompany CKD?

Pulmonary edema Uremic pleuritis Pneumonia

A patient who recently had a stroke is home after one month of rehabilitation. The home care nurse is concerned when the patient says (select all that apply): A. I just don't want to do anything anymore B My daughter visits twice a day and helps me C I just can't sleep at night, I wake up every hour. D I understand that my atrial fibrillation caused my stroke E I have no problem going up the stairs, why is everyone so worried?

A. I just don't want to do anything anymore C I just can't sleep at night, I wake up every hour. E I have no problem going up the stairs, why is everyone so worried?

Common psychosocial problems a patient may have post stroke include (select all that apply) a. depression. b. disassociation. c. sleep problems. d. intellectualization. e. denial of severity of stroke.

A. depression. C. sleep problems. E. denial of severity of stroke. Rationale: The patient with a stroke may have many losses, including sensory, intellectual, communicative, functional, role behavior, emotional, social, and vocational. Some patients have long-term depression, manifesting symptoms, including anxiety, weight loss, fatigue, poor appetite, and sleep problems. The time and energy needed to perform previously simple tasks can result in anger and frustration. Reactions vary considerably but may involve fear, apprehension, denial of the severity of stroke, depression, anger, and sorrow.

What are the nurses actions for stroke?

ABC's (airway, breathing, circulation) ICP measures-elevate bed to 30 degrees Elevate head of bed Carry out orders ASAP Neuro check and report changes DVT prevention (pneumo boots, PROM) Management of hypertension Seizure prophylaxis-siderails and bumpers.

What are tonic-clonic seizures?

AKA "grand-mal" seizures. The most common dramatic type of epileptic seizure. There is a sudden loss of consciousness, marked changes in tone body rigid and shaking, which lasts for <1 min (Tonic phase). Jerking occurs for 2-3 minutes (Clonic phase). Then flaccid and comatose which has a variable duration. Sometimes loss of bladder control or biting tongue.

A patient is having word finding difficulty and weakness in his right arm. What area of the brain is most likely involved? a. brainstem. b. vertebral artery. c. left middle cerebral artery. d. right middle cerebral artery.

C. left middle cerebral artery. Rationale: If the middle cerebral artery is involved in a stroke, the expected manifestations include aphasia, motor and sensory deficit, and hemianopsia on the dominant side and include neglect, motor and sensory deficit, and hemianopsia on the nondominant side.

What is the terrible triad?

Diabetes, heart disease, kidney disease

Dysphagia

difficulty swallowing

Ataxia

lack of muscle coordination

What is a seizure disorder/eplepsy?

A group of neurologic diseases marked by recurring seizures. 2 or more 24 hours a part without an underlying problem.

What are some of the symptoms of MS?

*Slurred speech, *Vision disturbances-diplopia Late symptom-paralysis Exacerbated by exposure to stress, fatigue and heat Fatigue and weakness Abnormal reflexes Motor dysfunction Sensory disturbances Impaired speech Urinary dysfunction Neurobehavioral

Describe Hyponatremia

- decreased plasma concentration of sodium low/fluid retained - inhibit ADH to increase the excretion of free water and increase plasma osmolality Contributes to edema, HTN, and heart failure

In which order do the events of antidiuretic hormone (ADH) secretion stimulated by plasma osmolarity occur? 1.Plasma osmolarity is increased .2. Osmoreceptors are activated. 3.Antidiuretic hormone (ADH) is released. 4.Water is reabsorbed from renal tubules. 5.Extracellular fluid is decreased.

-5,1,2,3,4 5.Extracellular fluid is decreased. 1.Plasma osmolarity is increased .2. Osmoreceptors are activated. 3.Antidiuretic hormone (ADH) is released. 4.Water is reabsorbed from renal tubules.

What is amaurosis fugax?

-Blindness in one eye -Sign of impending stroke. Emboli travel from the internal carotid artery to the ophthalmic artery, which impairs perfusion of the optic nerve and causes retinal dysfunction.

What is Broca's aphasia?

-Characterized by frontal lobe damage resulting in speech apraxia and agrammatism "walk dog, book 2 table" -The individual has good auditory comprehension but reading and writing are severely affected.

What is Wernicke's aphasia?

-Characterized by temporal lobe damage resulting in impaired auditory comprehension and feedback -Have fluent, well-articulated paraphasic speech (word substitution errors) Sentences that have no meaning.

What is the pathophysiology of CKD?

-Depends on the underlying disease reduced blood flow to kidneys and nephrons increased osmotic pressure to hyperglycemia -Loss of nephrons -Hypertrophy and higher workload -Predisposition to sclerosis -Reduced function -Additional nephron loss

What are the pathophysiology of risk factors for CKD in the glomerulus?

-Diabetes hyperglycemia, osmotic diuresis, compensatory cell growth to keep up with diuresis, elevated pressure in the glomerular capillaries, and leakage of albumin into the urine. -Hypertension Inadequate flow to glomerular capillaries, sclerosis and thickening of the capillary walls, remaining nephrons overwork to maintain GFR, high pressure weakens walls, small hemorrhages, and renal damage.

Describe the oliguric phase of AKI

-Lasts 8-14+ days - Urine output < 400ml/day - Increase BUN + Creatine -hypovolemia-fluid replacement helps -Metabolic acidosis-common acid-base imbalance -sodium balance-hyponatremia -potassium excess-driven out by sodium (arrhythmias-peaked T waves) -hematologic disorders -waste product accumulations -neurologic disorders Infection is the greatest cause of death

Describe the diuretic phase of AKI

-daily urine output is 1-3 liters per day but may reach 5L or more -may last 1-3 weeks -hypovolemia, hyponatremia (low sodium) may occur from massive fluid loss _caused by osmotic diuresis from the high urea concentration in the glomerular filtrate and inability of the tubules to concentrate the urine. -kidneys have recovered the ability to remove waste, but not the ability to concentrate urine -at the end of this phase patient's acid base, electrolyte and waste products values stabilize.

What is epilepsy?

A neurological disorder marked by chronic, recurrent pattern of unprovoked seizures. Abnormal electrical activity in the brain that disrupts normal function. symptoms include sensory disturbance, loss of consciousness, or convulsions.

What is dysarthria?

A problem with the muscular control of speech. The impairment may involve pronunciation, articulation, and phonation. It does not affect the meaning of communication or language comprehension. It does affect the mechanics of speech.

Which is the causative organism for syphilis? 1 Treponema pallidum 2 Campylobacter jejuni 3 Trichomonas vaginalis 4 Chlamydia trachomatis

1 Treponema pallidum

How does the nurse arrange the events of the positive feedback mechanism of estradiol chronologically?

1) The level of estradiol is increased during the menstrual cycle 2) Follicle-Stimulating Hormone production and release is increased 3) Estradiol levels are increased further due to FSH 4) Death of the follicle occurs 5) FSH levels in serum drop

What are the treatments for mineral and bone disorders?

First diagnose with bone biopsy, limit dietary phosphorus, give phosphate binders and supplement vitamin d

NIH stroke scale

1-4 minor stroke 5-15 moderate stroke 15-20 moderate to severe stroke 21-42 severe stroke

Diminished ability to concentrate urine, associated with aging of the urinary system, is attributed to a. a decrease in bladder sensory receptors b. a decrease in the number of functioning nephrons c. decreased function of the loop of Henle and tubules d. thickening of the basement membrane of Bowman's capsule

1. Correct answer: c Rationale: Older adults have decreased function of the loop of Henle and tubules, which results in the loss of normal diurnal excretory pattern because of a decreased ability to concentrate urine and because of less concentrated urine.

Formation of urine is done in the folloing:

1. Filtration at the glomerulus 2. Absorption into the peritubular capillaries 3. Reabsorption into the tubule for excretion in urine

Which type of hepatitis is transmitted by the fecal-oral route via contaminated food, water, or direct contact with an infected person? 1. Hepatitis A 2. Hepatitis B 3. Hepatitis C 4. Hepatitis D

1. Hepatitis A Rationale:1. The hepatitis A virus is in the stool of infected people for up to 2 weeks before symptoms develop2. Hepatitis B is spread through contact with infected blood and body fluids3. Hepatitis C is transmitted through contact with infected blood and body fluids4. Hepatitis D infection only causes infection in people who are also infected with Hepatitis B or C"

How is RLS diagnosed (the five criteria)?

1. Urge or desire to move legs/arms or other sensations 2. Begin or worsen during times of rest or inactivity. 3. Partially or totally relieved by activities such as stretching, walking exercising. 4. Worse or occurs in the evening or at night. 5. Are not solely due to another medical or behavioral problem.

What are the nursing diagnose for CKD?

Fluid imbalance Electrolyte imbalance Impaired nutritional status Difficulty coping

What are the two categories that seizures fall into?

Focal onset seizures Unknown onset seizures (limited to one hemisphere of the brain, impaired awareness/consciousness)

What is polyuria?

large volume of urine in a time period-diabetes, diabetes insipidus, chronic kidney disease, diuretics, excess fluid intake, obstructive sleep apnea.

How does the nurse arrange the events that take place during the promotion of glucose transportation into the cells through cell membranes? 1. Attachment of insulin to receptors 2. Secretion of proinsulin by beta cells 3. Storage of proinsulin in the pancreas 4. Transformation of proinsulin into active insulin

2. Secretion of proinsulin by beta cells 3. Storage of proinsulin in the pancreas 4. Transformation of proinsulin into active insulin 1. Attachment of insulin to receptors

A nurse is caring for a client with a diagnosis of type 1 diabetes who has developed diabetic coma. Which element excessively accumulates in the blood to precipitate the signs and symptoms associated with this condition?1.Sodium bicarbonate, causing alkalosis 2.Ketones as a result of rapid fat breakdown, causing acidosis 3.Nitrogen from protein catabolism, causing ammonia intoxication 4.Glucose from rapid carbohydrate metabolism, causing drowsiness

2.Ketones as a result of rapid fat breakdown, causing acidosis

How much fluid can the bladder hold?

200-250 mL of urine-causes moderate distention at a time. 400-600 mL - causes discomfort 600-1000 mL typical bladder capacity

What is hyperkalemia and the treatment?

A serious electrolyte disorder that can cause fatal dysrhythmias especially if levels reach 7-8 meq.L too much potassium, treatment IV glucose & insulin, calcium gluconate, sodium polystyrene (Kayexalate), Paritomer (Valtassa) The normal range for potassium 3.5-5.0

How does the parathyroid gland work in the urinary system?

It releases parathyroid hormone in response to low serum calcium levels Maintains serum calcium levels by causing increased tubular reabsorption of phosphate ions.

What is the ABCD scoring tool for TIA's

A risk assessment tool designed to predict the risk for stroke two days after a TIA calculates the score by adding up the points for the 5 factors, the higher the score the greater need for medical attention. Factors: ~Greater than 60 years old ~Systolic blood pressure greater than 140/90 ~Unilateral weakness with or without impaired speech ~Impaired speech without unilateral weakness ~Duration of greater than 60 minutes ~Diabetes

What is the role of the distal tubule?

It selectively secretes and absorbs different ions to maintain blood pH and electrolyte balance.

What are complications for drug therapy?

Kidney damage, drug toxicity

The patient with type 1 diabetes is having a seizure. Which medication would the nurse anticipate will be administered first? A IV dextrose solution B IV diazepam (Valium) C IV phenytoin (Dilantin) D Oral carbamazepine (Tegretol)

A IV dextrose solution Rationale: This patient's seizure could be caused by low glucose, so IV dextrose solution would be given first to stop the seizure. IV diazepam, IV phenytoin, and oral carbamazepine would be used first to treat seizures from other causes such as head trauma, drugs, and infections. These drugs will be tried if the IV dextrose is ineffective.

A patient with suspected renal insufficiency is scheduled for a creatinine clearance diagnostic test. Which instructions would be appropriate for the nurse to provide to the patient? A "Empty your bladder and discard the urine; then save all urine for 24 hours." B "Your blood creatinine level will be tested after you eat a high-protein meal." C "This test should not be performed if you have allergies to iodine or shellfish." D "A sterile container must be used to store the urine during the colle

A "Empty your bladder and discard the urine; then save all urine for 24 hours." Rationale: The patient should discard the first urination when this test is started. Urine should be saved from all subsequent urinations for 24 hours. Creatinine clearance testing does not involve the injection of contrast dye. A serum creatinine is determined during the 24-hour period and used in the calculation to determine creatinine clearance. Consumption of a high-protein meal is not indicated. Sterile containers would be indicated if cultures are performed to determine the presence of microorganisms.

A patient with a history of recurrent urinary tract infections has been scheduled for a cystoscopy. What teaching point would the nurse emphasize before the procedure? A "You might have pink-tinged urine and burning after your cystoscopy." B "You'll need to refrain from eating or drinking after midnight the day before the test." C "The morning of the test, you will drink some water that contains a contrast solution." D "You'll need a urinary catheter before the cystoscopy, and it will be in plac

A "You might have pink-tinged urine and burning after your cystoscopy." Rationale: Pink-tinged urine, burning, and frequency are common after a cystoscopy. The patient does not need to be NPO before the test, and contrast media is not needed. A cystoscopy does not always necessitate catheterization before or after the procedure.

The nurse is preparing a patient for an intravenous pyelogram (IVP). What is a priority action by the nurse? A Administer a cathartic or enema. B Assess patient for allergies to penicillin. C Keep the patient NPO for 4 hours pre-procedure. D Tell the patient a metallic taste be present during the procedure.

A Administer a cathartic or enema. Rationale: Nursing responsibilities in caring for a patient undergoing an IVP include administration of a cathartic or enema to empty the colon of feces and gas. The nurse will also assess the patient for iodine sensitivity; keep the patient NPO for 8 hours before the procedure; and advise the patient that warmth, a flushed face, and a salty taste during injection of contrast material may occur.

Which findings will the nurse expect when caring for a patient with chronic kidney disease (CKD)? (Select all that apply.) A Anemia B Dehydration C Hypertension D Hypercalcemia E Increased fracture risk F Elevated white blood cells

A Anemia C Hypertension E Increased fracture risk Rationale: When the kidney fails, erythropoietin is not excreted, so anemia is expected. Dehydration and hypercalcemia are not expected in chronic renal disease. Fluid volume overload with hypertension and hypocalcemia are expected. Hypocalcemia from chronic renal disease stimulates the parathyroid to release parathyroid hormone, causing calcium liberation from bones increasing the risk of pathological fracture. Although impaired immune function should be expected, elevated white blood cells would indicate inflammation or infection not associated with chronic renal failure itself but a complication.

A patient with stage 3 chronic kidney disease (CKD) is being taught about a low-potassium diet. The nurse recognizes the patient understands the diet when the patient selects which foods to eat? A Apple, green beans, and a roast beef sandwich B Granola made with dried fruits, nuts, and seeds V Watermelon and ice cream with chocolate sauce D Bran cereal with ½ banana and milk and orange juice

A Apple, green beans, and a roast beef sandwich Rationale: When the patient selects an apple, green beans, and a roast beef sandwich, the patient shows understanding of the low-potassium diet. Granola, dried fruits, nuts and seeds, milk products, chocolate sauce, bran cereal, banana, and orange juice all have high levels of potassium, at or above 200 mg per 1/2 cup.

A patient with stage 2 chronic kidney disease is scheduled for an outpatient diagnostic procedure using contrast media. Which priority action would the nurse perform? A Assess the patient's hydration status. B Insert a urinary catheter for the expected diuresis. C Evaluate the patient's lower extremities for edema. D Check the patient's urine for the presence of ketones.

A Assess the patient's hydration status. Rationale: Preexisting kidney disease is the most important risk factor for the development of contrast-associated nephropathy and nephrotoxic injury. If contrast media must be administered to a high-risk patient, the patient needs to have optimal hydration. The nurse should assess the hydration status of the patient before the procedure is performed. Indwelling catheter use should be avoided whenever possible to decrease the risk of infection.

A postoperative patient had a urinary catheter. Eight hours after catheter removal and drinking fluids, the patient has not been able to void. What test should the patient have first to assess for urinary retention? A Bladder scan B Cystometrogram C Residual urine test D Kidneys, ureters, bladder (KUB) x-ray

A Bladder scan Rationale: If the patient is unable to void, the bladder may be palpated for distention or percussed for dullness if it is full, or a bladder scan may be done to determine the approximate amount of urine in the bladder. A cystometrogram visualizes the bladder and evaluates vesicoureteral reflux. A KUB x-ray delineates size, shape, and positions of kidneys and possibly a full bladder. Neither of these would be useful in this situation. A residual urine test requires urination before catheterizing the patient to determine the amount of urine left in the bladder, so this assessment would not be helpful for this patient.

Which assessment findings would alert the nurse that the patient has entered the diuretic phase of acute kidney injury (AKI)? (Select all that apply.) A Dehydration B Hypokalemia C Hypernatremia D BUN increases E Urine output increases F Serum creatinine increases

A Dehydration B Hypokalemia E Urine output increases Rationale: The hallmark of entering the diuretic phase is the production of copious amounts of urine. Dehydration, hypokalemia, and hyponatremia occur in the diuretic phase of AKI because the nephrons can excrete wastes but not concentrate urine. Serum BUN and serum creatinine levels begin to decrease.

A patient was admitted 2 weeks ago after multiple traumatic injuries in a motor vehicle collision. The patient now has a serum creatinine at 3.9 mg/dL and blood urea nitrogen (BUN) of 100 mg/dL. Which medication, if ordered by the health care provider, would the nurse question? A Gentamicin B Nitrofurantoin C Acetaminophen D Morphine sulfate

A Gentamicin Rationale: Elevated serum creatinine and BUN indicate renal insufficiency or acute kidney injury. All medications should be evaluated for nephrotoxic potential. Many drugs are known to be nephrotoxic (see Table 49.3); gentamicin is a potential nephrotoxic agent.

The patient has rapidly progressing glomerular inflammation. Weight has increased and urine output is steadily declining. What is the priority nursing intervention? A Monitor the patient's cardiac status. B Teach the patient about hand washing. C Obtain a serum specimen for electrolytes. D Increase direct observation of the patient.

A Monitor the patient's cardiac status. Rationale: The nurse's priority is to monitor the patient's cardiac status. With the rapidly progressing glomerulonephritis, renal function begins to fail and fluid, potassium, and hydrogen retention lead to hypervolemia, hyperkalemia, and metabolic acidosis. Excess fluid increases the workload of the heart, and hyperkalemia can lead to life-threatening dysrhythmias. Teaching about hand washing and observation of the patient are important nursing interventions but are not the priority. Electrolyte measurement is a collaborative intervention that will be done as ordered by the health care provider.

A patient has been receiving scheduled doses of phenytoin and develops diplopia. Which additional findings would the nurse expect? A Nystagmus and confusion B An aura or focal seizure C Abdominal pain and cramping D Irregular pulse with palpitations

A Nystagmus and confusion

A patient was just diagnosed with Huntington's disease. The patient's adolescent child is upset about their parent's diagnosis. What is the nurse's best response? A Provide emotional and psychologic support. B Encourage him to get diagnostic genetic testing. C Explain that cognitive deterioration will be treated with counseling. D Teach that chorea and psychiatric disorders can be treated with haloperidol.

A Provide emotional and psychologic support. Rationale: The patient's son will first need emotional and psychologic support. He should be taught about diagnostic genetic testing but should decide for himself with a genetic counselor if and when he wants this done. The treatment plan for his father will be determined based on his father's needs.

In addition to urine function, the nurse recognizes that the kidneys perform numerous other functions important to maintaining homeostasis. Which physiologic processes are performed by the kidneys? (Select all that apply.) A Release of renin B Activation of vitamin D C Carbohydrate metabolism D Erythropoietin production E Hemolysis of old red blood cells (RBCs)

A Release of renin B Activation of vitamin D E Erythropoietin production Rationale: In addition to urine formation, the kidneys release renin to maintain blood pressure, activate vitamin D to maintain calcium levels, and produce erythropoietin to stimulate RBC production. Carbohydrate metabolism and hemolysis of old RBCs are not physiologic functions that are performed by the kidneys.

The nurse is caring for a patient after a right kidney biopsy. How would the nurse position the patient immediately after the procedure? A Right lateral side-lying position B Reverse Trendelenburg position C Supine with lower extremities elevated D High Fowler's position with arms supported

A Right lateral side-lying position Rationale: After a renal biopsy, a pressure dressing should be applied. The patient should be kept on the affected side for 30 to 60 minutes to apply additional pressure from the patient's own body weight and then on bed rest for 24 hours. High Fowler's position with arms supported is a position for a patient in respiratory distress. Reverse Trendelenburg position is used to maintain circulation to the legs in peripheral artery insufficiency. Supine with legs elevated puts excessive pressure on the diaphragm and should generally be avoided.

What are the urine studies for CKD?

A UA can detect RBC's WBC, protein, casts and glucose-persistent proteinuria is usually the first sign of kidney damage.

Which nursing intervention is a priority for a patient with multiple sclerosis (MS)? A Vigilant infection control and adherence to standard precautions B Careful monitoring of neurologic assessment and frequent reorientation C Maintenance of a calorie count and hourly assessment of intake and output D Assessment of blood pressure and monitoring for signs of orthostatic hypotension

A Vigilant infection control and adherence to standard precautions Rationale: Infection control is a priority in the care of patients with MS because infection is the most common cause of an exacerbation of the disease. Decreases in cognitive function are less likely, and MS does not typically result in malnutrition, hypotension, or fluid volume excess or deficit.

What is chronic pyelonephritis?

A continuous infection of the kidneys which leads to inflammation and fibrosis. There may be a loss of renal function and atrophy can occur.

The rapid response team is called to the cafeteria. They find a 22-year-old with a known seizure disorder having a tonic-clonic seizure. What actions will be taken during the seizure? (select all that apply) A establish IV access B insert an oral airway C gently hold the arms to prevent injury D adminster the patient's intranasal midazolam E turn the person on to their side and pad their head

A establish IV access D adminster the patient's intranasal midazolam E turn the person on to their side and pad their head Rationale: This patient has an established seizure disorder. Establish IV access. If they have midazolam nasal spray or midazolam autoinjector, this should be given. Nothing should be put in the patient's mouth during a seizure. No part of the body should be restrained. The patient should be turned on their side and their head protected from injury on the hard floor.

What are some of the medications for RLS?

Requip Dopamine Antiseizure medications Benzodiazepines Opioids

Which points must the nurse consider when planning nutrition support for patients with chronic kidney disease (select all that apply)? A sodium may be restricted B fluid is not usually restricted for patients on peritoneal dialysis C decrease fluid intake and a low potassium diet are needed for a patient on hemodialysis D decreased fluid intake and low potassium diet are needed for peritoneal dialysis E decreased fluid intake and a diet rich in protein-rich foods are part of a diet for a patient

A sodium may be restricted B fluid is not usually restricted for patients on peritoneal dialysis C decrease fluid intake and a low potassium diet are needed for a patient on hemodialysis Rationale: Water and any other fluids are not routinely restricted before Stage 5 end-stage renal disease (ESRD). Patients receiving hemodialysis have a more restricted diet than do patients receiving peritoneal dialysis. Patients receiving hemodialysis are taught about the need for a dietary restriction of potassium- and phosphate-rich foods. Patients receiving peritoneal dialysis may actually need potassium replacement because of the higher losses of potassium with peritoneal dialysis. Sodium and salt restriction is common for all patients with CKD. For those receiving hemodialysis, as their urinary output decreased, fluid restrictions are enhanced. Intake depends on the daily urine output. In general, 600 mL (from insensible loss) plus an amount equal to the previous day's urine output is allowed f

What are the seven functions of the kidneys (think a wet bed)

A-Acid base balance control W-Water balance E-Electrolyte balance T-Toxin & other waste product removal B-Blood pressure control E-Erythropoietin production D-activation of vitamin D

Stroke manifestations for the anterior cerebral artery:

motor and sensory deficit (contralateral) sucking or rooting reflex, rigidity, gait problem, loss of proprioception and fine touch.

What is hemiparesis?

paralysis/weakness of one side of the body

What is acute kidney injury?

Acute kidney injury is defined as the abrupt loss of kidney function over hours to days. ~most common cause-acute tubular necrosis ~ is diagnosed by an acute reduction in urine output or elevation in serum creatinine. ~potentially reversible ~cause of death-infection ~cause of AKI-surgery, illegal drugs, shock, sepsis, alcohol, medications, heart attack.

What are some contraindications for kidney transplant?

Advanced cancer Refractory untreated heart disease chronic reparatory failure extensive vascular disease chronic infection Unresolved psychosocial disorders HIV & Hepatitis are not contraindications

What are some non modifiable risk factors for stroke?

Age, gender, ethnicity, race, family history, and heredity.

What is status epilepticus?

An acute continuous seizure that lasts more than 5 minutes without returning to consciousness, long or occurring so close together that the patient cannot recover between them. Repetitive without periods of consciousness, more than 3 per hour. Is a serious neurologic emergency, and may be life-threatening. This may lead to - Hypoxia, trauma, hyperthermia, hypoglycemia, aspiration, and dehydration. Most often occur with children and older adults. The brain uses more energy than the body can supply, as neurons become exhausted they cease to function and brain damage can result.

What is an embolic stroke?

An embolus, blood clot or other debris circulating in the blood lodges in and occludes a cerebral artery.

What are some hematologic manifestations that accompany CKD?

Anemia Bleeding Infection

What are some gastrointestinal manifestations that accompany CKD?

Anorexia Nausea Vomiting GI bleeding Gastritis

What is anosognosia?

Anosognosia: Transient, severe form of neglect. Patient does not recognize the presence or severity of his paralysis. Patient is unaware of their condition.

What are some ways to manage seizures?

Antiepileptic drugs (Kepra, etc) Deep brain stimulation Responsive neurostimulation Neurosurgery Vagal Nerve Stimulation (VNS) Physical therapy Biofeedback

Categories of renal problems:

Anuric (no urine) less than 100 mL per day Oliguric (low urine) less than 400 mL per day Nonoliguric greater than 400 mL per day (better outcomes)

What are the types of aphasia?

Aphasia occurs when a stroke damages the dominate hemisphere of the brain. Receptive aphasia (loss of comprehension) Expressive aphasia (inability to produce language) Global aphasia (total inability to communicate)

What are the permanent accesses for hemodialysis?

Arteriovenous fistulas or grafts (can have temporary vascular access)

Hemodialysis procedure:

Assess fluid status Assess vascular access Heparin is added to prevent clotting Dialysate delivery and monitoring Lines primer Terminate by flushing Needles remove and pressure is applied

What is a clonic seizure?

Associated with repeated or rhythmic jerking muscle movements. These seizures usually affect the neck, face and arms.

Which statement about continuous ambulatory peritoneal dialysis (CAPD) would be most important when teaching a patient new to the treatment? A "Maintain a daily written record of blood pressure and weight." B "It is essential that you maintain aseptic technique to prevent peritonitis." C "You will be allowed a more liberal protein diet once you complete CAPD." D "Continue regular medical and nursing follow-up visits while performing CAPD."

B "It is essential that you maintain aseptic technique to prevent peritonitis." Rationale: Peritonitis is a potentially fatal complication of peritoneal dialysis, and thus it is imperative to teach the patient methods of prevention. Although the other teaching statements are accurate, they do not have the potential for morbidity and mortality that peritonitis does.

The home care nurse visits a patient receiving peritoneal dialysis. Which statement by the patient indicates a need for immediate follow-up by the nurse? A "Drain time is faster if I rub my abdomen." B "The fluid draining from the catheter is cloudy." C "The drainage is bloody when I have my period." D "I wash around the catheter with soap and water."

B "The fluid draining from the catheter is cloudy." Rationale: The primary manifestation of peritonitis is a cloudy peritoneal effluent. Blood may be present in the effluent of women who are menstruating, and no intervention is indicated. Daily catheter care may include washing around the catheter with soap and water. Drain time may be facilitated by gently massaging the abdomen.

The nurse teaches a 28-yr-old female patient who has migraine headaches about sumatriptan (Imitrex). Which statement by the patient requires clarification by the nurse? A "The injection might feel like a bee sting." B "This medicine will prevent a migraine headache." C "I can take another dose if the first does not work." D "This drug for migraine headaches could cause birth defects."

B "This medicine will prevent a migraine headache." Rationale: Sumatriptan is given to abort an ongoing migraine headache and is not used to prevent migraine headaches. When given as a subcutaneous injection, this drug may cause transient pain and redness at the injection site. This drug may be repeated after a specified time period if the first dose is not effective. This drug should be avoided during pregnancy and is classified as a Food and Drug Administration Pregnancy Risk Category C drug.

Which patient has the most significant risk factors for chronic kidney disease (CKD)? A A 50-yr-old white woman with hypertension B A 61-yr-old Native American man with diabetes C A 28-yr-old black woman with a urinary tract infection D A 40-yr-old Hispanic woman with cardiovascular disease

B A 61-yr-old Native American man with diabetes Rationale: The nurse identifies the 61-year-old Native American with diabetes as the most at risk. Diabetes causes about 50% of CKD. This patient is the oldest, and Native Americans with diabetes develop CKD 6 times more frequently than other ethnic groups. Hypertension causes about 25% of CKD. Hispanics have CKD about 1.5 times more than non-Hispanics. Blacks have the highest rate of CKD because hypertension is significantly increased in blacks. A UTI will not cause CKD unless it is not treated or UTIs occur recurrently.

A patient with end-stage renal disease (ESRD) secondary to diabetes has arrived at the outpatient dialysis unit for hemodialysis. Which assessments would the nurse perform as a priority before, during, and after the treatment? A Level of consciousness B Blood pressure and fluid balance C Temperature, heart rate, and blood pressure D Assessment for signs and symptoms of infection

B Blood pressure and fluid balance Rationale: Although all the assessments are relevant to the care of a patient receiving hemodialysis, fluid removal during the procedure will require monitoring blood pressure and fluid balance prior, during, and after.

The nurse observes a patient with Parkinson's disease rocking side to side while sitting in the chair. Which action by the nurse is most appropriate? A Provide the patient with diversional activities. B Document the activity in the patient's health record. C Take the patient's blood pressure sitting and standing. D Ask if the patient is feeling either anxious or depressed.

B Document the activity in the patient's health record. Rationale: Patients with Parkinson's disease are taught to rock from side to side to stimulate balance mechanisms and decrease akinesia.

A patient with type 2 diabetes and chronic kidney disease has a serum potassium level of 6.8 mEq/L. Which finding will the nurse monitor for? A Fatigue B Dysrhythmias C Hypoglycemia D Elevated triglycerides

B Dysrhythmias Rationale: Hyperkalemia is the most serious electrolyte disorder associated with kidney disease. Fatal dysrhythmias can occur when the serum potassium level reaches 7 to 8 mEq/L. Fatigue and hypertriglyceridemia may be present but do not require urgent intervention. Hypoglycemia is a complication related to diabetes control, not hyperkalemia. However, administration of insulin and dextrose is an emergency treatment for hyperkalemia.

A patient has sought care because of recent problems establishing and maintaining a urine stream and occasional pain with urination. How would the nurse document this finding? A Anuria B Dysuria C Oliguria D Enuresis

B Dysuria Rationale: Painful and difficult urination is characterized as dysuria. Whereas anuria is an absence of urine production, oliguria is diminished urine production. Enuresis is involuntary nocturnal urination.

During the oliguric phase of AKI the nurse monitors the patient for (select all that apply) A Hypotension B ECG changes C hypernatremia D pulmonary edema E urine with specific gravity

B ECG changes D pulmonary edema Rationale: The nurse monitors the patient in the oliguric phase of acute renal injury for the following: • Hypertension and pulmonary edema: When urinary output decreases, fluid retention occurs. The severity of the symptoms depends on the extent of the fluid overload. In the case of reduced urine output (i.e., anuria, oliguria), the neck veins may become distended with a bounding pulse. Edema and hypertension may develop. Fluid overload can eventually lead to HF, pulmonary edema, and pericardial and pleural effusions. • Hyponatremia: Damaged tubules cannot conserve sodium. Consequently, the urinary excretion of sodium may increase, which results in normal or below-normal serum levels of sodium. • Electrocardiographic changes and hyperkalemia: Initially, signs of hyperkalemia are apparent on electrocardiogram (ECG), including peaked T waves and a widening of the QRS complex. • Urinary specific gravity: Urinary specific gravity is fixed at about

A patient donated a kidney via a laparoscopic donor nephrectomy to a nonrelated recipient. The patient is having significant pain and refuses to get up to walk. How would the nurse respond? A Allow the patient to rest and try again tomorrow. B Encourage a short walk around the patient's room. C Have the transplant psychologist convince her to walk. D Tell the patient she is lucky she did not have an open nephrectomy.

B Encourage a short walk around the patient's room. Rationale: Because ambulating will improve bowel, lung, and kidney function with improved circulation, even a short walk with assistance should be encouraged after pain medication. The transplant psychologist or social worker's role is to determine if the patient is emotionally stable enough to handle donating a kidney; postoperative care is the nurse's role. Telling the patient she is lucky she did not have an open nephrectomy (implying how much more pain she would be having if it had been open) will not be beneficial to the patient or her postoperative recovery. Early ambulation should be encouraged, waiting until tomorrow is too long.

Diffusion, osmosis, and ultrafiltration occur in both hemodialysis and peritoneal dialysis. Which strategy is used to achieve ultrafiltration in peritoneal dialysis? A Increasing the pressure gradient B Increasing osmolality of the dialysate C Decreasing the glucose in the dialysate D Decreasing the concentration of the dialysate

B Increasing osmolality of the dialysate Rationale: Ultrafiltration in peritoneal dialysis is achieved by increasing the osmolality of the dialysate with additional glucose. In hemodialysis, the increased pressure gradient from increased pressure in the blood compartment or decreased pressure in the dialysate compartment causes ultrafiltration. Decreasing the concentration of the dialysate in either peritoneal or hemodialysis will decrease the amount of fluid removed from the blood stream.

A patient reports a throbbing headache with aura and photophobia. Which diagnosis would the nurse suspect? A Cluster headache B Migraine headache C Polycythemia vera D Hemorrhagic stroke

B Migraine headache Rationale: Although a headache may occur with any of these options, a migraine headache is the only one that has a throbbing headache with an aura (the photophobia). Headache from polycythemia vera is from erythrocytosis. The cluster headache pain is sharp and stabbing. The headache associated with a hemorrhagic stroke has a sudden onset and is not recurrent.

The nurse is caring for a patient with amyotrophic lateral sclerosis (ALS). What strategy would prevent a common cause of death for patients with ALS? A Reduce fat intake. B Reduce the risk of aspiration. C Decrease injury related to falls. D Decrease pain secondary to muscle weakness.

B Reduce the risk of aspiration. Rationale: Reducing the risk of aspiration can help prevent respiratory infections that are a common cause of death from deteriorating muscle function. Reducing fat intake may reduce cardiovascular disease, but this is not a common cause of death for patients with ALS. Decreasing injury related to falls and decreasing pain secondary to muscle weakness are important nursing interventions for patients with ALS but are unrelated to causes of death for these patients.

The nurse is caring for a patient who is in the oliguric phase of acute kidney disease. Which action would be appropriate to include in the plan of care? A Provide foods high in potassium. B Restrict fluids based on urine output. C Monitor output from peritoneal dialysis. D Offer high-protein snacks between meals.

B Restrict fluids based on urine output. Rationale: Fluid intake is monitored during the oliguric phase. Fluid intake is determined by adding all losses for the previous 24 hours plus 600 mL. Potassium and protein intake may be limited in the oliguric phase to avoid hyperkalemia and elevated urea nitrogen. Hemodialysis, not peritoneal dialysis, is indicated in acute kidney injury if dialysis is needed.

A nurse is caring for a patient with chronic obstructive pulmonary disease (COPD) admitted for pneumonia. What laboratory findings would be consistent with decreased kidney function? A Serum uric acid of 5.2 mg/dL B Serum creatinine 2.3 of mg/dL C Urine specific gravity of 1.040 D Blood urea nitrogen (BUN) of 10 mg/dL

B Serum creatinine 2.3 of mg/dL Rationale: An expected assessment finding related to decreased kidney function in the aging process is an increased serum creatinine. Other expected assessments include an elevated BUN and inability to concentrate urine (with urine specific gravity fixed at 1.010). Uric acid is used as a screening test for disorders of purine metabolism or kidney disease; values depend on renal function, rate of purine metabolism, and dietary intake of food rich in purines. Normal reference intervals: serum creatinine, 0.6 to 1.3 mg/dL; BUN, 6 to 20 mg/dL; urine specific gravity, 1.003 to 1.030; and serum uric acid, 2.3 to 6.6 mg/dL (female) or 4.4 to 7.6 mg/dL (male).

The nurse is performing an assessment for a patient and preparing to palpate the kidneys. How would the nurse position the patient for this assessment? A Prone B Supine C Seated at the edge of the bed D Standing, facing away from the nurse

B Supine Rationale: To palpate the right kidney, the patient is positioned supine, and the nurse's left hand is placed behind and supports the patient's right side between the rib cage and the iliac crest. The right flank is elevated with the left hand, and the right hand is used to palpate deeply for the right kidney. The normal-sized left kidney is rarely palpable because the spleen lies directly on top of it.

A patient with multiple sclerosis receives natalizumab every 6 months and modafinil and dalfampridine daily what assessment findings would indicate that the treatment plan was successful? Select all that apply: A blurred vision B improved walking C Decreased fatigue D no change in spacisitity E reduced exacerbation frequency

B improved walking C Decreased fatigue E reduced exacerbation frequency Rationale: Multiple Sclerosis patients have difficulty with vision, continence, constipation, spasticity, walking, fatigue, lack of concentration, and lack of coordination. MS has exacerbations and remissions leading to worse symptoms with each exacerbation. Natalizumab decreases the frequency and severity of exacerbations. Modafinil reduces fatigue, and dalfampridine enhances walking. Establishing a treatment plan with goals related to the reduction of these symptoms and reduction of exacerbations will increase the quality of life for a MS patient.

What is tubular secretion?

passage of a substance from the capillaries through the tubular cells into the lumen of the tubule

Which patient has the highest risk for having a stroke? a. An obese 45-year old Native American. b. A 65-yr-old black man with hypertension. c. A 35-yr-old Asian American women who smokes. d. A 32-yr-old white women taking oral contraceptives.

B. A 65-yr-old black man with hypertension. Rationale: Nonmodifiable risk factors for stroke include age (older than 65 years), male gender, and ethnicity (incidence is highest in Blacks; next highest in Hispanics, Native Americans/Alaska Natives, and Asian Americans; and next highest in white people). Modifiable risk factors for stroke include hypertension (most important), heart disease (especially atrial fibrillation), smoking, abdominal obesity, metabolic syndrome, and lack of physical exercise.

What is pneumaturia?

passage of urine containing gas (a fistula connections between bowel and bladder-gas forming UTI)

A patient with kidney disease has oliguria and a creatinine clearance of 40 mL/min. These findings most directly reflect abnormal function of a. tubular secretion b. glomerular filtration c. capillary permeability d. concentration of filtrate

B. glomerular filtration Rationale: The amount of blood filtered each minute by the glomeruli is expressed as the glomerular filtration rate (GFR). The normal GFR is about 125 mL/min.

Bladder training in a male patient who has urinary incontinence after a stroke includes a. limiting fluid intake. b. helping the patient to stand to void. c. keeping a urinal in place at all times. d. catheterizing the patient every 4 hours.

B. helping the patient to stand to void. Rationale: In the acute stage of stroke, the primary urinary problem is poor bladder control and incontinence. A bladder retraining program consists of (1) adequate fluid intake, with the greatest fluid intake between 7:00 AM and 7:00 PM; and (2) scheduled toileting every 2 hours with the use of a urinal, commode, or bathroom, (3) assisting with clothing and mobility; and (4) encouraging the usual position for urinating.

For a patient who is suspected of having a stroke, the most important piece of information that the nurse can obtain is a. time of the patient's last meal. b. time at which stroke symptoms first appeared. c. patient's hypertension history and management. d. family history of stroke and other cardiovascular diseases.

B. time at which stroke symptoms first appeared. Rationale: During initial evaluation, the most crucial point in the patient's history is the time since onset of stroke symptoms. If the stroke is ischemic, recombinant tissue plasminogen activator (tPA) must be given within 3 to 4½ hours of the onset of signs of ischemic stroke; tPA reestablishes blood flow through a blocked artery and prevents brain cell death in patients with acute onset of ischemic stroke.

What is BEFAST?

B: BALANCE: SUDDEN LOSS OF BALANCE E: EYES: LOSS OF VISION IN ONE OR BOTH EYES F: FACE: FACE looks uneven, droopy, and numbness A: ARM: ARM/LEG WEAK/HANGING DOWN S: SPEECH: SPEECH SLURRED OR TROUBLE SPEAKING/SEEMS CONFUSED T: TIME: Time is critical, CALL 911 NOW!

Discuss Diazepam (Diastat, Intensol, Valium) Nursing implications/patient education:

Black box warning related to the concomitant use of opioids. Contraindicated with any other respiratory depressants and in children less than 6 months old. Avoid the use of diazepam more than five episodes per month or one episode every five days. Women should be cautioned against getting pregnant or breast feeding during the use of this medication (previously a category Dper the US food and drug administration).

What are post renal kidney problems?

Bladder or prostate problems ~BPH (Benign Prostatic Hyperplasia) ~Calculi formation ~Bladder or prostate cancer ~Strictures ~Trauma (back, pelvis, perineum)

What are the manifestations of a lower urinary tract infection?

Bladder storage, bladder emptying. Symptoms include dysuria, frequency, urgency, suprapubic discomfort or pressure, may present with blood in the urine.

What is intracerebral hemorrhage?

Bleeding within the brain caused by rupture of vessel. Usually in the basal ganglia. The prognosis is poor. Hypertension is the most common cause. Manifestations include neurologic defects, headache, nausea, vomiting, decreased LOC, and hypertension.

Other aphasias

Result from damage to different language areas in the brain, some may have trouble repeating words and sentences even though they can understand the meaning of the word or sentence. May have trouble naming objects even though they know what the object is and what its use is.

The nurse prepares a patient for discharge after a cystoscopy. It is most important for the nurse to provide additional information in response to which patient statement? A "I should drink plenty of fluids to prevent complications." B "If my urine is cloudy, I should contact my health care provider." C "Bright red bleeding is normal for a few days after the procedure." D "Sitz baths and acetaminophen will help to reduce my discomfort."

C "Bright red bleeding is normal for a few days after the procedure." Rationale: Bright red bleeding after a cystoscopy is not normal and should be reported immediately. Other complications include urinary retention, bladder infection, and perforation of the bladder. Patients should drink plenty of fluids and expect burning on urination, pink-tinged urine, and urinary frequency. Warm sitz baths, heat, and mild analgesics may be used to relieve discomfort.

The nurse provides diet teaching to the in-home caregiver of a patient with Huntington's disease. The nurse would be most concerned if the caregiver makes which statement? A "Depression is common and may cause a decrease in appetite." B "If swallowing becomes difficult, a feeding tube may be needed." C "Calories should be restricted to prevent unnecessary weight gain." D "Muscles in the face are affected, and chewing may become impossible."

C "Calories should be restricted to prevent unnecessary weight gain." Patients with Huntington's disease may require 4000 to 5000 calories/day to maintain body weight. Weight loss occurs in patients with Huntington's disease because of choreic movements, difficulty swallowing, depression, and mental deterioration.

A patient tells the nurse that they have burning on urination, dysuria, and frequency. What is the best response by the nurse? A "Drink less fluid so you don't have to void so often." B "Take some acetaminophen to decrease the discomfort." C "Come in so we can check a clean-catch urine specimen." D "Avoid caffeine and spicy food to decrease inflammation."

C "Come in so we can check a clean-catch urine specimen." Rationale: The patient's symptoms are typical of a urinary tract infection. To verify this, a clean-catch urine specimen must be obtained for a specimen of urine to culture. Drinking less fluid will not improve the symptoms. Acetaminophen would not decrease the discomfort; an antibiotic would be needed. Avoiding caffeine and spicy food may decrease bladder inflammation but will not affect these symptoms.

The nurse provides information to the caregiver of a patient with epilepsy who has tonic-clonic seizures. Which statement by the caregiver indicates a need for further teaching? A "The jerking movements may last for 30 to 40 seconds." B "It is normal for a person to be sleepy after a seizure." C "I should call 911 if breathing slows during the seizure." D "Objects should not be placed in the mouth during a seizure."

C "I should call 911 if breathing slows during the seizure." Rationale: Caregivers do not need to call an ambulance or send a person to the hospital after a single seizure unless the seizure is prolonged, another seizure immediately follows, or extensive injury has occurred. Altered breathing is a manifestation of a tonic-clonic seizure. Contact emergency medical services (or call 911) if breathing stops for more than 30 seconds. No objects (e.g., oral airway, padded tongue blade) should be placed in the mouth. Lethargy is common in the postictal phase of a seizure. Jerking of the extremities occurs during the clonic phase of a tonic-clonic seizure. The clonic phase may last 30 to 40 seconds.

The nurse is caring for a group of patients on a medical unit. After receiving report, which patient would the nurse see first? A A 42-yr-old patient with multiple sclerosis who was admitted with sepsis B A 72-yr-old patient with Parkinson's disease who has aspiration pneumonia C A 38-yr-old patient with myasthenia gravis who declined prescribed medications D A 45-yr-old patient with amyotrophic lateral sclerosis who refuses enteral feedings

C A 38-yr-old patient with myasthenia gravis who declined prescribed medications. Rationale: Patients with myasthenia gravis who discontinue pyridostigmine (Mestinon) will develop myasthenic crisis. Myasthenia crisis results in severe muscle weakness and can lead to a respiratory arrest.

Which patient problem is the priority when caring for a patient with myasthenia gravis (MG)? A Acute confusion B Bowel incontinence C Activity intolerance D Disturbed sleep pattern

C Activity intolerance Rationale: The primary feature of MG is fluctuating weakness of skeletal muscle. Bowel incontinence and confusion are unlikely signs of MG. Although sleep disturbance is likely, activity intolerance is of primary concern.

When establishing a diagnosis of multiple sclerosis (MS), which diagnostic tests would the nurse expect? (Select all that apply.) A EEG B ECG C CT scan D Carotid duplex scan E Evoked response testing F Cerebrospinal fluid analysis

C CT scan E Evoked response testing F Cerebrospinal fluid analysis Rationale: No definitive diagnostic test exists for MS. Along with history and physical examination, CT scan, evoked response testing, cerebrospinal fluid analysis, and MRI are used to establish a diagnosis of MS. EEG, ECG, and carotid duplex scan are not used to diagnose MS.

The provider has decided to use renal replacement therapy to remove large volumes of fluid from a patient who is hemodynamically unstable in the intensive care unit (ICU). The nurse would expect which treatment to be used? A Hemodialysis (HD) three times per week B Automated peritoneal dialysis (APD) C Continuous venovenous hemofiltration (CVVH) D Continuous ambulatory peritoneal dialysis (CAPD)

C Continuous venovenous hemofiltration (CVVH) Rationale: CVVH removes large volumes of water and solutes from the patient over a longer period of time by using ultrafiltration and convection. HD three times per week would not be used for this patient because fluid and solutes build up and then are rapidly removed. With APD (used at night instead of during the day) fluid and solutes build up during the day and would not benefit this patient as much. CAPD will not as rapidly remove large amounts of fluid as CVVH can do.

During hemodialysis, the patient develops light-headedness and nausea. What would the nurse do first? A Give hypertonic saline. B Initiate a blood transfusion. C Decrease the rate of fluid removal. D Administer antiemetic medications.

C Decrease the rate of fluid removal. Rationale: The patient is having hypotension from a rapid removal of vascular volume. The rate and volume of fluid removal will be decreased, and 0.9% saline solution may be infused. Hypertonic saline is not used because of the high sodium load. A blood transfusion is not indicated. Antiemetic medications may help the nausea but would not help the hypovolemia.

The nurse is caring for an older adult patient taking bumetanide. What age-related changes does the nurse teach the patient they may experience? A Benign enlargement of prostatic tissues B Decreased sensation of bladder capacity C Decreased function of the loop of Henle D Less absorption in the Bowman's capsule

C Decreased function of the loop of Henle Rationale: Bumetanide (Bumex) is a loop diuretic that acts in the loop of Henle to decrease reabsorption of sodium and chloride. Because the loop of Henle loses function with aging, the excretion of drugs becomes less and less efficient. Thus, the circulating levels of drugs are increased and their effects prolonged. Benign prostatic tissue enlargement, decreased bladder sensation, and loss of concentrating ability do not directly affect the action of loop diuretics.

A patient is recovering in the intensive care unit (ICU) 24 hours after receiving a kidney transplant. What is an expected assessment finding during the earliest stage of recovery? Hypokalemia Hyponatremia Large urine output Leukocytosis with cloudy urine output

C Large urine output Rationale: Patients often have diuresis in the hours and days immediately after a kidney transplant. Electrolyte imbalances and signs of infection are unexpected findings that warrant prompt intervention.

Which characteristic would the nurse associate with a focal seizure? A The patient lost consciousness during the seizure. B The seizure involved both sides of the patient's brain. C The seizure involved lip smacking and repetitive movements. D The patient fell to the ground and became stiff for 20 seconds.

C The seizure involved lip smacking and repetitive movements. Rationale: Complex focal seizure is often characterized by lip smacking and automatisms (repetitive movements that may not be appropriate). Loss of consciousness, bilateral brain involvement, and a tonic phase are associated with generalized seizure activity.

The nurse conducts a home visit with a patient with Parkinson's disease taking levodopa/carbidopa daily. The patient has stopped eating meals with his family and has lost 3 pounds since the last visit a week ago. What action would the nurse take? A. plan 6 small high protein meals per day B. Provide information on a high fat ketogenic diet C evaluate their ability to eat, swallow and use of assistive devices. D. Collaborate with the HCP about every other day levodopa/carbidopa dosing.

C evaluate their ability to eat, swallow and use of assistive devices. Rationale: Patients with Parkinson's disorder have trouble with the coordinated act of eating, often require assistive devices to eat, and have difficulty swallowing. Diet is of major concern as malnutrition can occur without adequate calories and nutrient intake. The uncoordinated movements and tremors and slow eating can be embarrassing for the patient who may be isolating from the family. The nurse must determine if physical difficulty with the act of eating is causing the weight loss or if depression or another organic cause is to blame. Identifying the root of the problem would start with assessing the patient's abilities. Ketogenic diets are for headaches. Protein makes levodopa less effective, so protein should be limited to the evening meal. Reducing levodopa/carbidopa dosing will increase the tremors and bradykinesia.

When the patient is in the diuretic phase of AKI the nurse must monitor for serum electrolyte imbalances? A hyperkalemia and hyponatremia B hyperkalemia and hypernatremia C hypokalemia and hyponatremia D hypokalemia and hypernatremia

C hypokalemia and hyponatremia Rationale: In the diuretic phase of AKI, the kidneys have recovered the ability to excrete wastes but not the ability to concentrate urine. Hypovolemia and hypotension can result from massive fluid losses. Because of the large losses of fluid and electrolytes, monitor the patient for hyponatremia, hypokalemia, and dehydration.

What does the upper urinary system consist of?

Two kidneys and two ureters

A patient in the urgent care reports a stabbing headache around his left eye and nasal stuffiness. He says "this pain is horrible, it is worse than my heart attack last year." The provider orders sumatriptan nasal spray now for headache relief. The nurse would take what action? A. Administer a puff to each nostril now. B. Take vital signs prior to administration. C. Contact the provider about administering the medication to this patient. D. Teach the patient to self-administer the medication at

C. Contact the provider about administering the medication to this patient. Rationale The patient is experiencing a cluster headache. While triptans are appropriate for relieving an ongoing headache, sumatriptan causes vasoconstriction and is contraindicated for patients with an MI. The provider should be notified of this patient's history. Sumatriptan inhaler is given as one puff in one nare only. Verapamil would be used for the prevention of cluster headaches, sumatriptan is not prescribed to prevent cluster headaches. The nurse should take vital signs, but this patient should not take the medication, so the first action would be to verify the correct order.

The nurse explains to the patient with a stroke who is scheduled for angiography that this test is used to determine the a. presence of increased ICP. b. site and size of the infarction. c. patency of the cerebral blood vessels. d. presence of blood in the cerebrospinal fluid.

C. patency of the cerebral blood vessels. Rationale: Angiography provides visualization of cerebral blood vessels and can help estimate perfusion and detect filling defects in the cerebral arteries.

A patient having TIAs is scheduled for a carotid endarterectomy. The nurse explains that this procedure is done to a. decrease cerebral edema. b. reduce the brain damage that occurs during a stroke in evolution. c. prevent a stroke by removing atherosclerotic plaques blocking cerebral blood flow. d. provide a circulatory bypass around thrombotic plaques obstructing cranial circulation.

C. prevent a stroke by removing atherosclerotic plaques blocking cerebral blood flow. Rationale: In a carotid endarterectomy, the atheromatous lesion is removed from the carotid artery to improve blood flow.

What are the impacts of hypertension?

COronary artery disease, left ventricular hypertrophy, and heart failure. Brain-major risk for stroke Peripheral vasculature-speeds up atherosclerotic processes Kidney-renal disease, and kidney malfunction Eyes-retinal damage

What are the studies that confirm the diagnosis of stroke?

CT scan CT angiography (CTA) CT/MRI perfusion and diffusion imaging MRI Magnetic resonance angiography (MRA)

What are some Metabolic manifestations that accompany CKD?

Carbohydrate intolerance Hyperlipidemia

What cardiac assessments are required for stroke?

Cardiac markers (troponin, creatine kinase-MB) Chest X-ray Echocardiography (transthoracic, transesophageal) ECG

What factors manitain adequate blood flow to the brain?

Cardiac output, vasomotor tone, distribution to blood flow. Cardiac output must be reduced by one third before cerebral blood flow is reduced.

What the most common causes of death in a patient with CKD

Cardiovascular disease MI, heart failure, cardiomyopathy, stroke CVD and CKD are closely linked HTN is a cause and consequence of CKD

What are the studies that determine cerebral blood flow?

Carotid angiography Carotid duplex scanning Cerebral angiography Digital subtraction angiography Transcranial doppler ultrasonography

Discuss temporary vascular access for dialysis:

Catheter insertion of internal jugular or femoral vein when immediate access is needed. Double lumen-blood removal, blood return Risks:high infection, dislodgement, and malfunction.

What additional studies may help with stroke diagnosis and involvement?

Coagulation studies (prothrombin time, activated partial thromboplastin time) CBC with platelets Electrolyte panel with blood glucose Lipid profile Renal and hepatic studies.

What is the function of the tubules?

Collecting ducts that are responsible for the reabsorption of essential materials and excretion of the nonessentials. These functions are carried out by reabsorption and secretion.

What is the difference between complicated and uncomplicated UTI?

Complicated UTI"S occur in a patient with underlying disease with a structural or functional problem in the urinary tract. Uncomplicated UTI's occur in an otherwise normal urinary tract.

What are the urethras primary functions?

Control voiding Serve a a conduit for urine from the bladder to the outside of the body during voiding.

What is the best serum indicator for acute kidney injury?

Creatinine is not affected by other factors.

The bladder, urethra, and pelvic floor muscles form what?

Urethrovesical unit

What are the blood studies for CKD?

GFR-best indicator BUN-affected by diet, tissue breakdown, medications Creatinine-more sensitive than BUN-good indicator Elevate triglycerides- hyperinsulinemia stimulates hepatic production of triglycerides. Anemia-scant production of erythropoietin Metabolic acidosis-unable to excrete acid, unable to reabsorb sodium bicarbonate

A frail elderly patient with stage 3 chronic kidney disease is cared for at home by their family. The patient has a history of taking many over-the-counter medications. Which over-the-counter medications would the nurse teach the patient to avoid? A Aspirin B Acetaminophen C Diphenhydramine D Aluminum hydroxide

D Aluminum hydroxide Rationale: Antacids (that contain magnesium and aluminum) should be avoided because patients with kidney disease are unable to excrete these substances. Also, some antacids contain high levels of sodium that further increase blood pressure. Acetaminophen and aspirin (if taken for a short period of time) are usually safe for patients with kidney disease. Antihistamines may be used, but combination drugs that contain pseudoephedrine may increase blood pressure and should be avoided.

A patient with a 25-year history of type 1 diabetes is reporting fatigue, edema, and an irregular heartbeat. On assessment, the nurse notes newly developed hypertension and uncontrolled blood glucose levels. Which diagnostic study is most indicative of chronic kidney disease (CKD)? A Serum creatinine B Serum potassium C Microalbuminuria D Calculated glomerular filtration rate (GFR)

D Calculated glomerular filtration rate (GFR) Rationale: The best study to determine kidney function or CKD that would be expected in the patient with diabetes is the calculated GFR that is obtained from the patient's age, gender, race, and serum creatinine. It would need to be abnormal for 3 months to establish a diagnosis of CKD. A creatinine clearance test done with a blood sample and a 24-hour urine collection is also important. Serum creatinine is not the best test for CKD because the level varies with different patients. Serum potassium levels could explain why the patient has an irregular heartbeat. The finding of microalbuminuria can alert the patient with diabetes about potential renal involvement and potentially failing kidneys. However, urine albumin levels are not used for diagnosis of CKD.

Which patient diagnosis or treatment is most consistent with prerenal acute kidney injury (AKI)? A IV tobramycin B Incompatible blood transfusion C Poststreptococcal glomerulonephritis D Dissecting abdominal aortic aneurysm

D Dissecting abdominal aortic aneurysm Rationale: A dissecting abdominal aortic aneurysm is a prerenal cause of AKI because it can decrease renal artery perfusion and therefore the glomerular filtrate rate. Aminoglycoside antibiotic administration, a hemolytic blood transfusion reaction, and poststreptococcal glomerulonephritis are intrarenal causes of AKI.

A patient in the intensive care unit is receiving gentamicin for treatment of pneumonia from Pseudomonas aeruginosa. What assessment finding would the nurse report to the health care provider? A Decreased weight B Increased appetite C Increased urinary output D Elevated creatinine level

D Elevated creatinine level Rationale: Gentamicin can be toxic to the kidneys and the auditory system. The elevated creatinine level must be reported to the provider because it probably indicates renal damage. Other factors that may occur with renal damage would include increased weight and decreased urinary output. Many medications have side effects of anorexia.

A 28-year-old patient who's mother has Huntington disease is discussing family planning with the nurse. The patient says that they want to have a child but are concerned. What statement will the nurse make? A both parents must have the gene for your child to inherit this disease B if you have the Huntington mutation, you will not be able to have children. C only about 1 in 10 people who have the gene will pass it on to their children. D Genetic testing can be done to determine if you can pass th

D Genetic testing can be done to determine if you can pass this trait on to a child. Rationale: This patient is younger than the age when Huntington's disease manifests symptoms. Concern about passing the disorder can cause families to question their reproductive choices. The mutation is autosomal dominant and only 1 parent needs to carry it. 50% of their children can have the mutation. If they have the mutation, they will get the disorder. Since the patient has not shown symptoms and may not for many more years, they can be tested to see if they have the mutation and can pass it to any future children.

The nurse preparing to give a dose of calcium acetate to a patient with chronic kidney disease (CKD). Which laboratory result will the nurse monitor to determine if the desired effect was achieved? A Sodium B Potassium C Magnesium D Phosphorus

D Phosphorus Rationale: Phosphorus and calcium have inverse or reciprocal relationships, meaning that when phosphorus levels are high, calcium levels tend to be low. Therefore, administration of calcium should help to reduce a patient's abnormally high phosphorus level, as seen with CKD. Calcium acetate will not have an effect on sodium, potassium, or magnesium levels.

A patient with a diagnosis of Parkinson's disease (PD) is admitted to a long-term care facility. Which action would the health care team take to promote adequate nutrition? A Administer multivitamins every morning and with each meal. B Provide a diet that is low in complex carbohydrates and high in protein. C Give the patient with a pureed diet that is high in potassium and low in sodium. D Provide small, frequent meals throughout the day that are easy to chew and swallow.

D Provide small, frequent meals throughout the day that are easy to chew and swallow. Rationale: Nutrition support is a priority in the care of persons with PD. Patients may benefit from smaller, more frequent meals that are easy to chew and swallow. Multivitamins are not necessary at each meal. Vitamin and protein intake must be monitored to prevent interactions with medications. Introducing a minced or pureed diet is likely premature, and a low carbohydrate diet is not indicated.

The factor related to cerebral blood flow that most often determines the extent of cerebral damage from a stroke is the a. O2 content of the blood. b. amount of cardiac output. c. level of CO2 in the blood. d. degree of collateral circulation.

D. degree of collateral circulation. Rationale: The extent of the stroke depends on the rapidity of onset, size of the lesion, and presence of collateral circulation.

What does the lower urinary system consist of?

Urinary bladder and urethra.

When caring for a patient during the oliguric phase of acute kidney injury (AKI), which nursing action is appropriate? A Weigh patient three times weekly. B Increase dietary sodium and potassium. C Provide a low-protein, high-carbohydrate diet. D Restrict fluids according to previous daily loss.

D Restrict fluids according to previous daily loss. Rationale: Patients in the oliguric phase of AKI will have fluid volume excess with potassium and sodium retention. Therefore, they will need to have dietary sodium, potassium, and fluids restricted. Daily fluid intake is based on the previous 24-hour fluid loss (measured output plus 600 mL for insensible loss). The diet also needs to provide adequate, not low, protein intake to prevent catabolism. The patient should also be weighed daily, not just three times each week.

A patient reports acute, severe, renal colic pain in the lower abdomen. The nurse suspects that an obstruction is most likely at which area? A Kidney B Urethra C Bladder D Ureterovesical junction

D Ureterovesical junction Rationale: The ureterovesical junction is the narrowest part of the urethra and easily obstructed by urinary calculi. With a stone in the kidney or at the ureteropelvic junction, the pain may be dull costovertebral flank pain. Stones in the bladder do not cause obstruction or symptoms unless they are staghorn stones. The urethra seldom has obstruction related to stones.

The nurse obtained a urine specimen from a patient. What result would the nurse recognize as abnormal? A pH of 6.0 B Amber yellow color C Specific gravity of 1.025 D White blood cells (WBCs) 9/hpf

D White blood cells (WBCs) 9/hpf Rationale: Normal WBC levels in urine are below 5/hpf, with levels exceeding this indicative of inflammation or urinary tract infection. A urine pH of 6.0 is average; amber yellow is normal coloration, and the reference range for specific gravity is 1.003 to 1.030.

A renal stone in the pelvis of the kidney will alter the function of the kidney by interfering with the a. structural support of the kidney b. regulation of the concentration of urine c. entry and exit of blood vessels at the kidney d. collection and drainage of urine from the kidney

D collection and drainage of urine from the kidney Rationale: the outer layer of the kidney is the cortex, and the inner layer is the medulla. The medulla consists of a number of pyramids. The apices (tops) of these pyramids are called papillae, through which urine passes to enter the calyces. The minor calyces widen and merge to form major calyces, which form a funnel-shaped sac called the renal pelvis. The minor and major calyces transport urine to the renal pelvis, from which it drains through the ureter to the bladder.

The nurse using RIFLE to determine the early stage of AKI evaluates the patients: A blood pressure and urine osmolality B fractional excretion of urinary sodium C estimation of GFR with MDRD equation D serum creatinine or urine output from baseline

D serum creatinine or urine output from baseline Rationale: The RIFLE classification describes the stages of AKI. RIFLE standardizes the diagnosis of AKI. Risk (R) is the first stage of AKI, followed by injury (I), which is the second stage, and then increasing in severity to the last or third stage of failure (F). The 2 outcome variables are loss (L) and end-stage renal disease (E). The first 3 stages are characterized by the serum creatinine level and urine output.

Information provided by the patient that would help distinguish a hemorrhagic stroke from a thrombotic stroke includes a. sensory changes. b. a history of hypertension. c. presence of motor weakness. d. sudden onset of severe headache.

D. sudden onset of severe headache. Rationale: A hemorrhagic stroke usually causes a sudden onset of symptoms, which include neurologic deficits, headache, nausea, vomiting, decreased level of consciousness, and hypertension. Ischemic stroke symptoms may progress in the first 72 hours as infarction and cerebral edema increase.

What causes MS (multiple sclerosis)

Damage or breakdown (demyelination) of myelin sheath in the brain and spinal cord.

What is oliguria?

Decreased urine output in a time period (usually 24 hours)-severe dehydration, shock, transfusion reaction, kidney disease.

What are the Modifiable causes of CKD?

Diabetes Cardiovascular disease HTN Obesity Poverty Dyslipidemia Exposure to nephrotoxins Chronic UTI's

What are intrarenal kidney problems?

Direct damage to the kidney tissue, cause impaired nephron function. Substances may block-acute tubular necrosis ~interstitial nephritis-allergies, antibiotics (sulfonamides, rifampin) NSAIDS, ACE inhibitors, infections, bacteral (viral or fungal) ~Nephrotoxic injury-chemicals, contrast dye, drugs (aminoglycosides, gentamicin, amikacin, amphotericin B), blood transfusion reaction, crush injury. ~Other toxemia of pregnancy, systemic lupus erythematosus.

What are some ways to manage MS

Disease-modifying drugs Immunomodulators (interferon, teriflunomide) Corticosteroids Adrenocorticotropic hormone Chronic symptom management: Spasticity- Baclofen (Ileorenal) & Dantrolene (Atrium)-give before PT Fatigue-Amantadine (Symmetrel) Depression Bladder-anticholinergics Dystonia-Carbamazepine (Tegretol) Pain syndromes Carbamazepine (Tegretol) & Phenytoin (Dilantin)

What are the risks of AV fistulas and grafts?

Distal ischemia (steal syndrome) pain to the distal access site numbness or tingling of fingers poor capillary refill aneurysms Remember no BP or veinpunctures or iv lines in these areas, prevent infection and clotting.

Stroke manifestations for the middle cerebral artery:

Dominant side aphasia motor and sensory deficit, hemianopsia Non dominant side neglect, motor and sensory deficit hemianopsia.

What is the function of the efferent arteriole?

Drains blood away from the glomerulus, it maintains blood pressure and glomerular filtration rate.

What are some AKI diagnoses?

Electrolyte imbalance Fluid imbalance Risk for infection (#1 cause of death) Anxiety

What is the treatment for status epilepticus?

Establish airway, labs, oxygen, establish IV, anticonvulsants wich include Lorazepam-Ativan, Phenytoin-Dilantin, Diazepam-Valium (IV) Prevent injury-seizure pads can occur with cessation of anticonvulsive med or alcohol intake.

What are the treatments for RLS?

Exercise Good sleep habits Avoiding or limiting caffeine Avoid anti histamines Apply a heating pad, cold compress, massage Soak in a warm tub Magnesium supplements Reduce stress

How is anemia treated?

Exogenous erythropoietin (EPO) given IV or subcutaneously 2-3 times a week, reduce the need for blood transfusions.

What are the early warning signs for CKD?

Fatigue, weakness difficulty or pain on urination foamy urine-albumin reduces surface tension pink, dark urine increase in need to urinate and nocturia puffy eyes swelling increased thirst

What are the functions of the kidneys?

Filter the blood and maintain the body's internal homeostasis.

Describe hypermagnesemia

High serum magnesium level usually occurs only with an increase in mg intake accompanied by renal insufficiency or failure - A pt with CKD who ingests products containing mg (e.g. Maalox, milk of mag) will have a problem with excess mg - depresses neuromuscular and CNS functions -Initial clinical manifestations of a mildly elevated serum mg concentration include lethargy, nausea, and vomiting, absent reflexes, respiratory failure---as the levels of serum mg increase, deep tendon reflexes are lost, followed by somnolence, and then respiratory and, ultimately, cardiac arrest can occur

What are some Endocrine/reproductive manifestations that accompany CKD?

Hyperparathyroidism Thyroid abnormalities Amenorrhea Erectile dysfunction

What are some cardiovascular manifestations that accompany CKD?

Hypertension Heart failure Coronary artery disease Pericarditis Peripheral artery disease

What are some modifiable risk factors for stroke?

Hypertension, cardiac disease (a-fib and heart disease), diabetes mellitus, obesity, diet, high cholesterol, use of oral contraceptives with high dose of estrogen, cigarette smoking, alcohol abuse.

What are some ocular manifestations that accompany CKD?

Hypertensive retinopathy

What is urinary retention?

Inability to urinate even though the bladder contains excess amount of urine-after pelvic surgery, childbirth, catheter removal, anesthesia, urethral stricture or obstruction, neurogenic bladder.

Spatial perceptual problems after a stroke fall into four categories:

Incorrect perception of self and illness-deny illness ont recognize own body parts-parietal lobe damage. Spatial neglect-neglects the imput from the affected side, has trouble with orientation and distance judgement. Agnosia-the inability to recognize an object by sight, touch or hearing Apraxia-The inability to carry out learned sequential movements on command.

Describe hypernatremia

Increased serum sodium concentration sodium high/fluid loss indicates that body fluids are too concentrated (osmolality is too high) and exceeds 145-hyperosmolar

What is polynephritis?

Inflammation or infection of the kidneys, renal parenchyma and collecting system, including the renal pelvis. Most commonly caused by bacterial infection.

What is a thrombotic stroke?

Injury to a vessel wall and formation of a blood clot. The blood vessel becomes narrowed and if it becomes occluded an infarction will occur.

What is transluminal angioplasty?

Insertion of balloon to open stenosed artery to improve blood flow via femoral artery catheterization.

Which is the target tissue for the parathyroid hormone? Intestines All body cells Mammary glands sympathetic effectors

Intestines

What are generalized onset seizures?

Involve both hemispheres of the brain. Characterized by bilateral synchronous neural discharges from the onset of the seizure. Impaired consciousness/awareness for a few seconds to several minutes. Either motor or non-motor. Movements are bilateral. No loss of consciousness.

What two factors occur during a stroke?

Ischemia-inadequate blood flow Hemorrhage-bleeding into the brain which results in death of brain cells.

What are some characteristics of MS?

It is a chronic, progressive neurologic disease of the CNS of unknow etiology and uncertain trajectory. Autoimmune Characterized by the occurrence of small patches of demyelination of the white matter of the optic nerve, brian and spinal cord. Onset is slow and gradual There are exacerbations and remissions Most common CNS disease among young adults, and third leading cause of disability in the USA. Northern, caucasians

What is a transcranial doppler ultrasound?

It is a non invasive study that measures the velocity of blood flow in the major cerebral arteries. It is effective in detecting microemboli and vasospasms.

How can a ketogenic diet help control seizures?

It is a special diet that is high fat low carbohydrate diet that helps control seizures in some people. It makes ketones that pass into the brain.

How is hypertension an indicator for CKD?

It is both a cause and symptom.

What is chronic kidney disease?

It is characterized by the gradual, irreversible loss of functional nephrons. ~can happen over the years-asymptomatic ~most common cause-diabetic nephropathy ~diagnosed by GFR less than 60 for 3 months or more ~is progressive and irreversible ~cause of death-cardiovascular disease-MI

What is carotid duplex scanning used for?

It is used to detect the cause of a stroke and stratify patients for medical management or carotid intervention.

What is the role of the ascending loop of Henle?

It reabsorbs sodium and chloride from the filtrate into the interstitial fluid.

What is the role of the collecting duct?

It reabsorbs solutes and water from the filtrate

What are ways that MS is diagnosed?

MRI, Electrophoresis of CSF, visual, auditory and somatosensory evoked potentials. To be diagnosed with MS the patient must have: 1. Evidence of at least two inflammatory demyelinating lesions in at least two different locations within the CNS. 2. Damage or an attack occurring at different times (usually one month or more apart) 3. Other possible diagnoses ruled out.

Tension type headache

Mild to moderate bilateral headache with a sensation of a tight band or pressure around the head. Can be recurrent.

What are the nursing assessments for MS?

Motor strength, coordination and gate. Cranial nerve assessment (fifth-trigeminal most involved) Evaluate elimination function (bowel and bladder) Explore coping, and its effect on activity, sexual function, and emotional adjustment. Assess patient and family coping, support systems, and available resources.

What are some patient teachings for seizure meds?

No grapefruit juice report visual changes do not stop med on own a.s it causes worse problems

What are the three phases of acute kidney injury?

Oliguric, diuretic, recovery

Describe hyperphosphatemia as a result of hypocalcemia:

PTH stimulates bone demineralization releasing calcium from the bones, phosphate is also released leading to high serum phosphate levels, decreased phosphate excretion from the kidneys and further decreases serum calcium levels. More at risk for fractures and osteoporosis

What are some appearance and objective manifestations of CKD?

Pale, yellowish grey skin tone dry skin and poor turgor Edema Bruises Poor concentration, slow mentation Osteodystrophy, osteoporosis

What are some ways to check motor function on someone with MS?

Palpate the temporal and masseter muscles as the patient clenches teeth. Try to separate the jaw by pushing down on the chin. Should have same jaw strength bilaterally.

What are the stroke manifestations for right side of the brain stroke?

Paralysis on the left side of the body-hemiplegia Left sided neglect Spatial perceptual deficits Tends to deny or minimize problems Rapid performance, short attention span Impulsive, safety problems Impaired judgement Impaired time concepts.

What are the stroke manifestations for left side of the brain stroke?

Paralysis on the right side of the body-hemiplegia Impaired speech/language aphasias Impaired right/left discrimination Slow performance, cautious Aware of deficits, depression, anxiety Impaired comprehension related to language, math. Memory problems

What are some peripheral neuropathy manifestations that accompany CKD?

Paresthesia Restless leg syndrome

What is renal failure?

Partial or complete impairment of kidney function that results in an inability to excrete waste products and water. ~affects all body systems ~Treatments and dietary changes are challenging ~impacts lifestyle, occupation, family relationships and self image.

What are the main drugs for treating tonic clonic and focal onset seizures?

Phenytoin, carbamazepine, (Tegretol) and divalporex (Depakote)

What is the post seizure period called?

Post ictal period, people are usually groggy, weak and sleepy

Restless leg syndrome (RLS) aka Willis Ekbom disease

Primary cause- a CNS disorder-idiopathic-most common Secondary cause-anemia, pregnancy, renal failure, stress, diet, Parkinson's medication or other medication side effects. More common in women May be genetic causes a strong urge to move associates with other sensations and diseases Associated with some medication such as antidepressants and allergy meds.

What are some nutritional therapies for people with CKD?

Protein-normal protein for most people, restriction for CKD patients. Fluid restriction-Hemodialysis Sodium and potassium restriction-Hemodialysis, Peritoneal dialysis may need to take potassium Phosphate restriction-foods high in phosphorus are also high in protein.

What is a strong marker of for the progression of chronic kidney disease?

Proteinuria, it also is a marker of increased cardiovascular morbimortality. Filtration of albumin by the glomerulus is followed by tubular reabsorption and the resulting albuminuria reflects the combined contribution of these two processes. Healthy kidneys do not let albumin pass from blood to urine.

What are some integumentary manifestations that accompany CKD?

Puritis Ecchymosis Dry, scaly skin Uremic frost-rarely

What is the role of the proximal convoluted tubule?

Reabsorbs ions, water, and nutrients and removes toxins, and adjusts filtrate pH about filters about 80%

What are the primary functions of the kidneys?

Regulate the volume and composition of extracellular fluid Excrete waste products from the body The kidneys function to control blood pressure Make erythropoietin Activate vitamin D Regulate acid base balance

What are the manifestations of an upper urinary tract infection?

Renal parenchyma, pelvis and ureters. Usually causes fever, chills, and flank pain. No typical systemic manifestations.

Nursing assessments and interventions post seizure:

Seizure pads in bed and maintain patent airway Loosen tight neckwear Turn to side lying position Talk to them and offer reassurance Medication information Health promotion Document seizure activity, aura, movement, present of lip smacking, swalloging (automatisms), incontinence, duration, conscious/unconscious, behaviour post seizure.

What are tonic seizures?

Seizures that cause stiffening of the muscles, these seizures usually affect muscles in the back, arms, and legs and may cause the patient to fall to the ground.

What is urosepsis?

Severe systemic infection that occurs when an infection starts in the urinary tract and spreads into the bloodstream.

Staging of chronic kidney disease

Stage 1-GFR less than 90-CVD risk reduction, slow progression Stage 2 GFR 60-89 estimation of progression Stage 3a GFR 45-59 evaluation & treatment of complications Stage 3b GFR 30-44 more aggressive treatment Stage 4 GFR 15-29 prep for RRT dialysis or transplant Stage 5 GFR less than 15 or dialysis RRT if uremia, pt desires treatment, necessary to maintain life

How is dyslipide

Statins such as lipitor, especially for those with diabetes

What are atonic seizures?

Sudden drop to the ground as a result of loss of muscle control, patient suddenly collapses or falls down.

What is a sudep?

Sudden unexpected death in epilepsy is a fatal complication of epilepsy. It is defined as the sudden and unexpected, non-traumatic and non-drowning death of a person with epilepsy.

What is a seizure?

Sudden, abnormal excessive electrical discharge of neurons in the brain.

What are some of the factors that affect blood flow to the brain?

Systemic blood pressure, cardiac output, blood viscosity

What are some of the symptoms of seizures?

Temporary confusion Staring-blank look Uncontrollable jerking movements of arms and legs Loss of consciousness or awareness Psychic symptoms such as fear, anxiety or Deja vu

What are some ways to check sensory function?

Test light sensation with a cotton ball over the forehead (ophthalmic) Cheeks (maxillary) Chin (mandibular) Sensation should be intact and equal bilaterally.

What are myoclonic seizures?

Usually appear as sudden brief jerks or twitches of the arms and legs.

As the GFR decreases as a result of CKD what happens?

The BUN and serum creatinine levels increase, the BUN increases not only from kidney disease, but also protein intake, fever, steroids, catabolism, creatinine is a better indicator.

What arteriole brings blood into the glomerulus?

The afferent arteriole, it is a branch of the renal artery that brings blood to the glomerulus.

What are prerenal kidney problems?

The blood flow to the kidney is compromised, reducing systemic circulation and causing a reduction of blood flow. ~Decreased cardiac output such as MI, heart failure, dysrhythmia, and shock. ~Decreased peripheral vascular resistance-septic shock, anaphylaxis, neurologic injury ~Hypovolemia-burns, dehydration, GI losses, hemorrhage, excessive diuresis. ~Decreased renovascular blood flow-embolism, renal artery thrombosis.

what is cerebral autoregulation?

The brain is protected for changes in mean systemic arterial blood pressure, this helps keep blood flow at a constant.

The amount of blood is filtered per minute by the glomeruli is called?

The glomerular filtration rate (GFR)

Describe Hypocalcemia

The kidneys are unable to convert vitamin D to its active form resulting in decreased serum calcium levels (low calcium).

What is stress incontinence?

The loss or leaking of urine during exercise, sneezing, laughing, coughing, or when lifting something heavy. Weakness of sphincter control, lack of estrogen, urinary retention.

What is the functional unit of the kidney?

The nephron

peritoneal dialysis (PD)

The removal of wastes, electrolytes, and fluids from the body using the peritoneum as a dialysis membrane. Glucose-passive movement. Omentum is the filter.

What is a hemoragic stroke?

The result of bleeding into the brain tissue itself (intracerebral oor intraparenchymal hemorrhage) into the subarachnoid space or ventricles. Treatment includes; Surgical decompression if indicated, clipping or coiling of aneurysm.

What is the area where the ureter joins the renal pelvis?

The ureteropelvic junction

What are two key functions of the distal convoluted tubules?

They are the final regulation water balance and acid base balance.

What is the function of osmoreceptors in the anterior hypothalmus?

They detect a decrease in plasma osmolality. They send neural input to cells in the hypothalamus and act to inhibit ADH secretion.

How is a person's affect changed by a stroke?

They may have a hard time controlling their emotions, emotional responses may be exaggerated or unpredictable. Depression and feelings due to changes in the body image and loss of function.

What are focal seizures?

They start in one hemisphere of the brain Can spread to the other hemisphere. (previously called partial or partial focal seizures) No loss of consciousness. Alters emotions changes the way that things look or smell, taste, and sound. some jerking, tingling. dizziness. Focal with impaired awareness-change in loss of consciousness.

What does RIFLE stand for?

This is the staging system for acute kidney injury -A good predictor of patient outcome, GFR, and urine output are criteria used to stage AKI Risk Injury Failure Loss End-stage renal disease -relies on changes in serum creatinine or glomerular filtration rates and or urine output. -Allows the identification of AKI patients hospitalized in numerous settings, enables monitoring of severity

What are some diagnostic studies for AKI?

Thorough history Serum creatinine, bun, electrolytes (happens late in AKI) Urinalysis Renal ultrasound Renal scan CT scan masses and vascular Renal biopsy-interrenal causes

What is global aphasia?

Total inability to communicate, non fluent aphasia, results from damage to extensie portions of the language areas of the brain.

What are some treatment options for renal replacement therapy when conservative therapy is not effective for AKI?

Treatment options: -Peritoneal dialysis (not frequently used) -Intermittent hemodialysis (HD)-emergent therapy -Continous renal replacement therapy (CRRT) cannulation of artery and vein continuous for 24 hours

What is the glomerulus?

a collection of up to fifty capillaries, they unite in the efferent arteriole.

During physical assessment of the urinary system, the nurse a- performs fist percussions to detect tenderness in the flank area b- expects a dull percussion sounds when 100ml of urine is present in the bladder c- percusses above the symphysis pubis to determine the level of urine in the bladder d- palpates the lower pole of the right kidney as smooth mass that descends on expiration

a performs fist percussions to detect tenderness in the flank area Rationale: Tenderness in the flank area may be detected by fist percussion. This technique is performed by striking the fist of one hand against the dorsal surface of the other hand, which is placed flat along the posterior CVA margin. Normally this should not elicit pain. A bladder does not percuss until it contains at least 150 mL of urine. If the bladder is full, dullness is heard above the symphysis pubis. A distended bladder may be percussed as high as the umbilicus. Occasionally the lower pole of the right kidney is palpable. It palpates as a smooth, round mass that descends on inspiration.

What is an aura?

a subjective sensation that precedes a seizure; it could be auditory, visual, or motor.

What is a TIA stroke?

a transient episode of neurologic dysfunction, caused by focal brain, spinal cord or retinal ischemia, without acute infarction of the brain. Symptoms last less than one hour. TIA's can progress into larger strokes. The signs and symptoms depend on the blood vessels involved and the area of the brain that is ischemic. Treatment may include a low dose aspirin for the rest of their life.

Normal findings expected by the nurse on physical assessment of the urinary system include ( select all that apply) a- nonpalpable bladder b- nonpalpable left kidney c- auscultation of renal artery bruit d- no CVA tenderness elicited by a kidney punch e- full bladder percusses a dullness above the symphysis pubis

a- nonpalpable bladder b- nonpalpable left kidney d- no CVA tenderness elicited by a kidney punch e- full bladder percusses a dullness above the symphysis pubis Rationale: In the physical assessment of the urinary system, normal findings include no CVA tenderness and nonpalpable kidneys and bladder. If the bladder is full, dullness is heard above the symphysis pubis.

A kidney transplant recipient complains of having fever, chills, and dysuria over the past 2 days. what is the first action that the nurse should take? a. assess temperature and initiate workup to rule out infection b. reassure the patient that this is common after transplant c. provider warm cover for the patient and give 1 g tylenol orally d. notify the nephrologist that the patient has developed symptoms of acute rejection

a. assess temperature and initiate workup to rule out infection Rationale: The nurse must be astute in the observation and assessment of kidney transplant recipients because prompt diagnosis and treatment of infections can improve patient outcomes. Fever, chills, and dysuria indicate an infection. Assess the temperature. The patient should undergo diagnostic testing to rule out an infection.

The nurse identifies a risk for urinary calculi in a patient who relates a past health history that includes a. dehydration b. hyperaldosteronism c. serotonin deficiency d. adrenal insufficiency

a. dehydration Rationale: Dehydration may contribute to UTIs, stone formation, and kidney failure. Large intake of specific foods, such as dairy products or foods high in proteins, may lead to stone formation.

What is hematuria?

presence of blood in the urine. Cancer of the genitourinary tract, bloody dysstasias, kidney disease, UTI, stones in the kidney or ureter, anticoagulants.

Nurses can screen patients at risk for developing chronic kidney disease. Those considered to be at increased risk include (select all that apply) a. older black patients. b. patients more than 60 years old. c. those with a history of pancreatitis. d. those with a history of hypertension. e. those with a history of type 2 diabetes.

a. older black patients. b. patients more than 60 years old. d. those with a history of hypertension. e. those with a history of type 2 diabetes. Rationale: Risk factors for CKD include diabetes, hypertension, age older than 60 years, cardiovascular disease, ethnic minority (e.g., Black, Native American), family history of CKD, and exposure to nephrotoxic drugs.

A patient is admitted to the hospital with chronic kidney disease. the nurse understands that this condition is characterized by a. progressive irreversible destruction of the kidneys b. a rapid decrease in urine output with an elevated BUN c. an increasing creatinine clearance with a decrease in urine output d. prostration, somnolence, and confusion with coma and imminent death

a. progressive irreversible destruction of the kidneys Rationale: Chronic kidney disease (CKD) involves progressive, irreversible loss of kidney function. A focus on stages 1 through 4 before the need for dialysis (stage 5) includes the control of hypertension, hyper parathyroid disease, CKD-MBD, anemia, and dyslipidemia.

An ESRD patient receiving hemodialysis is considering asking a relative to donate a kidney for transplantation. In assisting the patient to make a decision about treatment, the nurse informs the patient that a. successful transplantation usually provides better quality of life than that offered by dialysis b. if rejection of the transplant occurs, no further treatment for the renal failure is available c. hemodialysis replaces the normal functions of the kidneys, and patients do not have to live

a. successful transplantation usually provides better quality of life than that offered by dialysis Rationale: Kidney transplantation is extremely successful, with 1-year graft survival rates of about 90% for deceased donor organs and 95% for live donor organs. An advantage of kidney transplantation over dialysis is that it reverses many of the pathophysiologic changes associated with renal failure when normal kidney function is restored. It also eliminates the dependence on dialysis and the need for the accompanying dietary and lifestyle restrictions. Transplantation is less expensive than dialysis after the first year.

What is hypertension and the treatment?

abnormally high blood pressure, treated conservatively, then two medications to reach target PB and ACE and an ARB to preserve renal function by decreasing proteinuria and delaying CKD progression.

Where is aldosterone released from?

adrenal cortex and acts on the distal tubule to cause reabsorption sodium and water. In exchange for sodium, potassium ions are excreted.

What is the role of the descending loop of Henle?

aquaporins allow water to pass from the filtrate into the interstitial fluid. Fursonemide is affected here

hemodialysis (HD)

procedure for removing impurities from the blood because of an inability of the kidneys to do so. Use pressure to take off fluid. A filter is used.

A diagnostic study that evaluates renal blood flow, glomerular filtration, tubular function, and excretion is a(n) a- IVP b- VCUG c- renal scan d- loopagram

c- renal scan Rationale: A renal scan evaluates the anatomic structures, perfusion, and function of kidneys. The scan shows the location, size, and shape of the kidneys and helps assess blood flow, glomerular filtration, tubular function, and urinary excretion. In some facilities, a numerical value ("split" renal function) may be assigned (i.e., percent contributed by each kidney).

What happens in the proximal convoluted tubule?

reabsorption of water, ions, and all organic nutrients

to assess the patency of a newly place AV graft for dialysis, the nurse should (SATA) a. monitor the BP in the affected arm b. irrigate the graft daily with low-dose heparin c. palpate the area of the graft to feel a normal thrill d. listen with a stethoscope over the graft to detect a bruit e. frequently monitor the pulses and neurovascular status distal to the graft

c. palpate the area of the graft to feel a normal thrill d. listen with a stethoscope over the graft to detect a bruit e. frequently monitor the pulses and neurovascular status distal to the graft Rationale: A thrill can be felt on palpation of the area of anastomosis of the arteriovenous graft, and a bruit can be heard with a stethoscope. The bruit and thrill are created by arterial blood rushing into the vein. The BP should not be taken in the arm with the AV graft.

What is a migraine headache?

recurring headache characterized by unilateral throbbing pain

What are the main drugs for treating focal seizures?

carbamazepine, lamotrigine (Lamictal), levetiracetam (Keppra), pregabalin (Lyrica)-for focal awareness or impaired awareness seizures not controlled with one medication.

What may develop over time to compensate for decrease in cerebral blood flow?

collateral circulation, the blood vessels in the brain make alternative routes for the blood to flow.

On reading the urinalysis results of a dehydrated patient, the nurse would expect to find a- a pH of 8.4 b- RBC's of 4 hpf c- color: yellow, cloudy d- specific gravity of 1.035

d- specific gravity of 1.035 Rationale: Normal specific gravity of urine is 1.005 to 1.030; the concentrating ability of the kidneys is maximal in producing morning urine (1.025 to 1.030). A high urinary specific gravity value indicates dehydration.

What are the three processes of MS?

inflammation first demyelination gliosis of the CNS-occurs when the body creates larger glio cells which cause scarring in the brain and impacts how the body works, can cause necrosis. Disorder of nerve impulses

What is cystitis?

inflammation of the bladder

What is urethritis?

inflammation of the urethra

What is enuresis?

involuntary nocturnal urination-lower urinary tract disorder.

What are the main drugs for treating generalized on set non motor and myoclonic seziures?

ethosuximide (Zarontin), divalproex and clonazepam (Klonopin)

What is the role of the glomerulus?

filters small solutes from the blood

What are some causes of siezures?

genetic head trauma brain conditions ex: tumor infectious disease (meningitis or aids) prenatal injury-hypoxic birth injury metabolic disorders development disorders idiopathic lack of folic acid can lead to tuberal sclerosis

What is the filtering unit of the kidney?

glomerulus

What are some complications of hemodialysis?

hypotension hypovolemia light-headed (TX decreases volume flow) nausea, seizures, vision changes, chest pain muscle cramps decreased blood pressure loss of blood hepatitis C Hep B-low incidence, give vaccine

What is azotemia?

increase in nitrogenous waste products in blood

Acid-Base Regulation

involves reabsorbing and conserving most of the bicarbonate and secreting excess hydrogen ions.

What is a cluster headache?

severe, unilateral headache localized to periorbital/temporal area accompanied by lacrimation, rhinorrhea, ptosis, myosis, nasal congestion, eyelid edema -Occurs in clusters meaning 1-8 daily attacks lasting 15-90 minutes for 4-6 weeks... followed by pain free interval 3-6 months

What is an ischemic stroke?

strokes caused by blockage or interrupted blood supply to the brain. Ischemic strokes can be caused by a thrombus or embolus. Treatment: tissue plasminogen activator (tPA) IV Tenecteplase or intraarterial, endovascular therapy. Clot retrieval/evacuation of blood, should be on normal saline for the first 24 hours. Speech and swallow evaluation before feeding, DVT prophylaxis, ICP-Mannitol to reduce edema BP needs to be managed before TPA given Seizure prophylaxis (Keppra)

What is the nephron?

structural and functional unit of the kidney

What is tubular reabsorption?

the passage of a substance from the lumen of the tubules through the tubule cells and into the capillaries

What is a subarachnoid hemorrhage?

there is intracranial bleeding into the cerebrospinal fluid space between the arachnoid and pia mater membranes on the surface of the brain. Usually caused by a rupture of a cerebral aneurysm. Warning signs include: loss of consciousness/some people do not lose consciousness. Focal neurologic deficits, nausea, vomiting, seizures and stiff neck. Cerebral vasospasm is common.

Describe the recovery phase of acute kidney injury:

~Begins with GFR increases (allows BUN & creatinine to decrease) increased kidney function allows waste to be removed. ~Kidney function may take up to 12 months to stabilize ~Some patients do not recover and progress to end stage renal disease (ESRD) ~Older adults are less likely to have a complete recovery of kidney function.

What are the different types of status epilepticus?

~Convulsive status epilepticus-most common form, with prolonged or repeated tonic-clonic or other motor seizures. ~Non-convulsive status epilepticus-repeated focal impaired awareness seizures. ~Refractory status epilepticus-continuous seizure activity despite administration of first and second-line therapy-high risk for mortality and neurologic damage. ~Super refractory-continuous or recurs 24 hours or more after starting anesthesia treatment. ~Subclinical seizures-a form of status epilepticus in which a sedated patient seizes but no external signs because of sedative use. Most often from tonic clinic seizures. Common at night.

What is care relate to AKI?

~Managing fluid volumes determine the adequate intravascular volume and cardiac output for perfusion to kidneys diuretics may be given-furosemide/lasix, bumetanide/bumex, osmotic diuretics/Mannitol fluid restriction (add up losses from day before) ~Hyperkalemia (most serious) Insulin and sodium bicarb are used to promote the transient shift of potassium to cells, dietary restriction of potassium and hemodialysis, oral suspension that binds potassium in GI tract patiromer/altessa ~Nutritional therapy sodium & potassium limited based on plasma levels, sodium-edema, HTN and heart failure. Carbs and fat to prevent ketosis from fat breakdown and glucogenesis. High protein may be bad.

How is epilepsy diagnosed?

~Medical History ~Tests: CBC, serum electrolytes, serum Ca++, ,liver and kidney function, urinalysis, drug levels, skull & chest x-ray to look for abnormalities, lumbar puncture, brain studies (CT and MRI), electroencephalography ~Imaging (EEG) to look at the electrical activity of the brain and abnormalities (should be done within 24 hours of suspected seizure) ~Clinical Observation: motor function, sensory function, neuropsychological eval ~Notes: accurate descriptions and video recordings help diagnose & classify types of seizures, CT, MRI, MRA, MRS, PET scan.


Conjuntos de estudio relacionados

Unit 6: Team Communication (Questions)

View Set

100 Rules of Will Construction in Texas

View Set